CFP Investing

¡Supera tus tareas y exámenes ahora con Quizwiz!

If an American investor holds foreign securities and is concerned about exchange rate risk, that investor may hedge by

buying a futures contract for delivery of dollars.

The Sharpe ratio

does not assume the portfolio is well diversified.

The daily limit of a commodity futures contract is the maximum

price increase or decrease relative to the settlement price the previous day.

Select a benefit of investing in foreign markets.

Diversification

Assuming Mary earned a 3% return from dividend reinvestment, a 2% return from capital gain reinvestment, and a 9% return from share price appreciation on her mutual fund, calculate her total return.

The answer is 14%. Total return on a stock or mutual fund may be thought of as the sum of the capital appreciation/depreciation on the underlying principal of the investment and any income or earnings generated from that investment. Therefore, Mary's total return equals 14% (3% + 2% + 9%).

Identify the entity that issues guaranteed investment contracts (GICs).

Insurance companies

What is the taxable equivalent yield on a municipal bond with an 8.75% return for an investor in the 24% marginal tax bracket?

8.75% ÷ (1 - 0.24) = 8.75% ÷ 0.76 = 11.51%

What type of index is the Dow Jones Industrial Average (DJIA)?

Price-weighted

Your client owns a small-cap fund and wants to compare its performance to an appropriate benchmark. You would advise him to choose which benchmark?

Russell 2000

Select the term that measures how far the actual outcomes of a probability distribution deviate from the arithmetic mean

Skewness

On January 1, 20XX, one U.S. dollar could buy 105 Japanese yen; on December 31, 20XX, one U.S. dollar could buy 121 Japanese yen. Which one of these statements best illustrates what has happened to the U.S. dollar during year 20XX?

The U.S. dollar was revalued.

JK Mutual Fund has a return of 15.5% and a beta of 1.1. If the market return is 14%, and the risk-free rate is 4%, what is the alpha for JK Mutual Fund?

The answer is +0.50. Calculate alpha: 15.5 - [4 + (14 - 4)1.1] = +0.50.

Keegan is an analyst for Global Growth and Income Mutual Fund. For the past five years, the fund has returned -20%, 17%, 5%, 15%, and -7%, respectively. Calculate the geometric mean of these returns.

The answer is 1.0018%. The geometric mean return is calculated as follows: PV = -1 FV = (1 - 0.20)(1 + 0.17)(1 + 0.05)(1 + 0.15)(1 - 0.07) = 1.0511 N = 5 Solve for I/YR = 1.0018, or 1.0018%

Brenda is interested in calculating the inflation-adjusted rate of return of a recent investment. Assuming the after-tax return on her investment is 6.25% and the inflation rate is 5%, calculate the inflation-adjusted rate of return.

The answer is 1.19%. The inflation-adjusted rate of return is calculated as follows: [(1.0625 ÷ 1.05) − 1] × 100 = 1.19%

The yield to maturity on a zero-coupon bond ($1,000 par value) currently selling at $677 and maturing in four years is approximately

The answer is 10.00%. Solve for yield to maturity: END Mode, 2 P/YR PV = −677 FV = 1,000 PMT = 0 4, DOWNSHIFT, N = 8 Solve for I/YR = 10% (rounded).

A $1,000 U.S. Treasury note maturing in eight years is selling for $938.12. The semiannual coupon payment is $35. What is the yield to maturity (YTM) for the note?

The answer is 8.06%. The note's YTM is computed as follows: END Mode, 2 P/YR PV = −938.12 PMT = 35 FV = 1,000 8, DOWNSHIFT, N = 16 Solve for I/YR = 8.06%

The current yield of an 8% coupon bond, maturing in five years, and selling currently for $850 is

The answer is 9.41%. Current yield = annual interest payment ÷ current market price = $80 ÷ $850 = 0.09412, or 9.41%.

The Allegro Mutual Fund has an alpha of +1.50, a Sharpe ratio of 0.44, and an R2 with the Russell 2000 of 0.45. The Moderato Mutual Fund has an alpha of +0.40, a Sharpe ratio of 0.48, and an R2 with the Russell 2000 of 0.52. Which fund should be chosen and why?

The answer is the Moderato Fund because has a higher Sharpe ratio. In order to use alpha (which uses beta in the formula), beta needs to be a reliable number. This can be determined by R2, which gives the level of systematic risk. The R2 for both funds is low (0.45 and 0.52), meaning beta, and formulas using beta, should not be used. You want an R2 of 0.70 or higher in order to use beta. This then leaves you with Sharpe, and the Moderato Fund has the highest Sharpe ratio of the two.

Bond ABC is selling at par, offers an 8% coupon, and matures in 20 years. The bond has a call feature that allows the issuer to call the bond after 10 years at a price of $1,050. Which of the following statements explains the relationship between the bond's yield to call (YTC) and yield to maturity (YTM)?

The answer is the YTC for Bond ABC is 8.33%, which is more than Bond ABC's YTM. Because Bond ABC is selling at par, the YTM is equal to the coupon rate of 8%. END Mode, @ P/YR PV = -1,000 FV = 1,050 10, DOWNSHIFT, N = 20 PMT = 8% × 1,000 ÷ 2 = 40 Solve for I/YR = 8.33%.

The risk associated with the amount of debt a company has issued is

financial risk. Financial risk is the risk related to the amount of debt a company has. Business risk is the risk associated with the nature of the business. Interest rate risk is the risk that as interest rates increase, bond prices decrease. Systematic risk is the risk associated with all factors affecting all comparable investments.

If the intrinsic value of a call option is $3, which of these statements is CORRECT?

he option is in-the-money.

To evaluate the performance of a portfolio manager, you should calculate the portfolio's

time-weighted return.

Beverly owns two stocks with a correlation coefficient of zero. Which of these is CORRECT?

These stocks will move independently of each other.

Calculate the intrinsic value of a call option that trades at $4, with an exercise price of $35, and has a current underlying stock price of $33.

$0; because $33 is less than $35. The call premium is $400 ($4 × 100 shares); therefore, the market feels the stock price will appreciate before the call expires.

LAC Corporation stock is currently trading for $180 per share. If the company institutes a 3-for-2 stock split, calculate the company's stock price following the stock split.

$120

Chelsea purchases a warrant for $2 per share that gives her the right to buy 80 shares of XYZ stock at $20 per share for a period of five years from date of purchase. Assume XYZ stock goes up to $25 per share after three years and Chelsea exercises the warrant. What profit does she make on the 80 shares?

$240: profit = (gain on stock - cost of warrant) × number of shares; ($5/share - $2/share) × 80 shares = $240

What is the coefficient of variation for an investment with a standard deviation of 8.65%, an expected return of 11.5%, and a beta of 1.25?

0.7522. CV = standard deviation of asset ÷ expected return of asset, 8.65% ÷ 11.5% = 0.7522.

Acme Electric Company announces a cash dividend of $0.50 per share on August 5, to be paid on September 20, the payable date. The company also announces that the record date will be August 25. Bob Johnson purchases 100 shares of Acme on August 24. Based on this information, choose the CORRECT statement regarding the dividend payment.

Bob will not receive the dividend, because he did not purchase the shares before the ex-dividend date.

Identify which of these statements regarding rights and warrants is CORRECT. I.Rights provide current common stockholders with the ability to retain their ownership percentage when new shares of stock are issued. II.Warrants are typically attached to new bond issues to attract investors.

Both I and II

Rhett recently purchased a bond with attached warrants that afford him the opportunity to participate in the appreciation of the underlying stock. Which of the following statements correctly describes warrants? I.Warrants are customized to fit the needs of the issuing corporation. II.Warrants typically have a maturity date of several years.

Both I and II

Which of these statements regarding a real estate mortgage investment conduit (REMIC) is CORRECT? I.A REMIC is a self-liquidating, flow-through entity that invests exclusively in real estate mortgages or mortgage-backed securities. II.A REMIC terminates when the mortgages that constitute the investment of the REMIC are repaid.

Both I and II

Which of these represents the best reason to include real estate as part of an investment portfolio?

Low correlation between real estate and equity investments

Which of the following would be an appropriate index to track an investment in an international developed markets mutual fund?

MSCI EAFE

Identify which of these is NOT a source of systematic risk. A)Reinvestment rate risk B)Market risk C)Business risk D)Purchasing power risk

business risk. Business risk is a type of unsystematic risk. Unsystematic risks only affect one company, country, or sector and its related securities.

An investor fearing a bear market would hedge his or her position by

buying a put.

Alex owns a put option with an exercise price of $51. The underlying stock is currently trading at $48 in the secondary market. Assuming the put option is currently trading at $6 and has three months until expiration, calculate the intrinsic value of the put option.

$300

An investor is researching a mutual fund. Last year this fund had a total return of 12% when the stock market had a 10% return. This fund has a beta of 1.2 and a standard deviation of 14%. The risk-free rate of return is 5%. What is the Sharpe ratio for this fund?

0.50Therefore, (0.12 - 0.05) ÷ 14 = 0.07÷0.14 = 0.50.

A client has a $1.2 million portfolio consisting of these four stocks: 1.$300,000ABC @1.1 beta 2.$225,000RTR @0.7 beta 3.$405,000XYZ @0.3 beta 4.$270,000PDQ @1.3 beta What is the beta of the portfolio as a whole?

0.8

Gary Stevens would like to know the weighted beta for his portfolio. He owns 100 shares of ACE common stock with a beta of 1.1 and total current market value of $5,000; 400 shares of BDF common stock with a beta of 0.70 and total current market value of $8,000; and 200 shares of GIK common stock with a beta of 1.5 and total current market value of $10,000. What is the overall weighted beta coefficient for Gary's portfolio?

1.13

Assuming JHG and DSA stocks have standard deviations of 6.23% and 10.78%, respectively, and a correlation coefficient of 0.17, calculate the covariance between the two stocks.

11.42. The covariance between the two stocks is a positive 11.42 (6.23 × 10.78 × 0.17). Covariance measures the extent to which two variables move together, either positively (together) or negatively (opposite).

Brantley recently purchased a new video game system on his credit card. Assuming the nominal annual percentage rate (APR) is 14.95% (compounded daily), calculate the effective annual rate (EAR).

16.12%. The effective annual rate on his credit card is 16.12%, calculated as follows: EAR = [1 + (0.1495 ÷ 365)]365 - 1 = 0.1612, or 16.12%.

our client, Jackson, is considering adding XYZ Mutual Fund to his portfolio. The fund has a correlation coefficient of 0.55 with the S&P 500, and he wants to know how much systematic risk the fund has when compared to this benchmark. You would advise him that the percentage of systematic risk is

30%. You have been provided with the correlation coefficient (R) and what you need is the coefficient of determination (R2). Keystrokes are 0.55, DOWNSHIFT, "+" key = 0.3025. This means that 30% of the price movement of the fund is explained by the S&P 500, and the other 70% is not. Stated another way, there is 30% systematic risk, and 70% unsystematic risk.

Jefferson originally purchased 100 shares of XYZ stock for $45 per share. The stock is currently trading at $60 per share. The stock paid dividends of $2 per share in year 1 and $2.30 per share in year 2 (all paid at year end). If Jefferson has held the stock for two years, what is his holding period return?

42.9%. This is calculated as: [($60 - $45) + $2 + $2.30] ÷ $45 = 42.9%.

The Gemini Fund has a correlation coefficient of 0.80 with the S&P 500 Index. How much of the price movement of the Gemini Fund can be explained by the S&P 500 Index?

64%. The correlation coefficient (R) has been given, so it needs to be squared (R2) in order to come up with the coefficient of determination (0.802 = 0.64).

Portfolio A has a standard deviation of 55%, and the market has a standard deviation of 40%. The correlation coefficient between Portfolio A and the market is 0.50. Calculate the percentage of total risk that is unsystematic.

75%. The coefficient of determination explains the percentage of change in the dependent variable that can be explained by changes in the independent variable. Therefore, 25% (0.50 × 0.50) of returns are explained by changes in the market. To determine the percentage of returns that are explained by unsystematic risk, subtract the systematic risk from 1. Therefore, the return explained by unsystematic risk is (1 − 0.25) = 0.75, or 75%.

The annual returns of the ABC fund have been +12%. -4%, and +7%. What is the standard deviation of the fund's returns?

8.19%

Jim is a paper maker who purchases lumber from tree farmers around his state. Which of these hedge positions should Jim consider if he is concerned with rising lumber prices?

A long hedge; Jim should buy lumber futures contracts to protect against rising lumber prices.

LTD Inc., a candy manufacturer, is considering the use of sugar futures contracts because sugar is a major ingredient in the manufacturing process. What type of hedge position should LTD take in the sugar futures market, and why?

A long hedge; the company should purchase sugar futures contracts because it is hedging against higher sugar prices.

Greg buys a call option on LMN Corporation's stock for an option premium of $1.50. Which of these statements is CORRECT? I.Greg hopes that the price of LMN stock will decline. II.Greg's maximum loss on the option is $150.

II only

Which of the following regarding mutual fund performance is CORRECT? I.Past performance is a reliable predictor of future performance. II.Past performance offers some indication as to the competency of fund managers.

II only

Mike expects a certain stock to significantly rise in value in the near future. He is expecting a bond to mature in two months and does not want to miss out on any appreciation on the stock while waiting for the funds to become available. Which of these option strategies should be recommended to Mike?

Buy a call option

Which of the following is a potential pitfall of mutual fund investing?

Buying a fund ranked number one if the fund is misclassified

What kind of investment company has no provision for redemption of outstanding shares?

Closed-end company

Your clients have invested in a variety of mutual funds, including foreign country funds. You expect the U.S. dollar to strengthen against the Japanese yen over the next year. Which of these actions would be most appropriate?

Hedge by taking a short position in yen futures.

Advantages of unit investment trusts include which of these? I.Stable periodic income II.Diversification III.Active management of the portfolio

I and II

Which of these statements concerning portfolio diversification is CORRECT? I.By increasing the number of securities in a portfolio, the total risk would be expected to fall at a decreasing rate. II.Total risk is reduced as diversification is increased. III.The benefits of diversification are not realized until at least 30 individual securities are included in the portfolio. IV. Diversification reduces the portfolio's expected return because diversification reduces a portfolio's total risk.

I and II. Studies have shown that an investor only needs about 15-20 assets to fully diversify a portfolio. The main attraction of diversification is the reduction of risk without an accompanying loss of return.

Which of these statements are CORRECT of mutual fund dividend distributions? I.The fund pays dividends from net investment income. II.A single taxpayer may exclude $100 worth of dividend income from taxes annually. III.An investor is liable for taxes on distributions whether a dividend is a cash distribution or is reinvested in the fund. IV. An investor is not liable for taxes if he or she automatically reinvests distributions.

I and III

Which of the following statements are true regarding risk and return for tangible investments? I.Unsystematic risk tends to be high with most forms of tangible assets. II.High cash flows from collectibles create annual taxable events. III.The illiquid nature of the investments makes them suitable for income-seeking investors. IV.The capital gains rate on collectibles is greater than that of other investments.

I and IV

Why would financial planners recommend their clients purchase natural resources for their investment portfolios? I.Diversification II.Low degree of risk III.Positive correlation with other financial assets IV.Pass through of certain tax benefits including depletion

I and IV

Identify the CORRECT statements regarding warrants. I.Warrants give the owner the right to purchase a specified number of shares for a specified period at a specified price. II.Warrants are typically written with a maturity date of nine months. III.Warrants must include standardized terms required by the Options Clearing Corporation. IV. Warrants are issued by a corporation rather than written by an individual.

I and IV. Warrants typically have a maturity date of several years, not months, and are customized to fit the needs of the issuing corporation.

A client of yours, George, wants to maximize his return on an intermediate-term bond that he plans to hold until maturity. You have gathered information on the following two bonds, both of which have a $1,000 par value. Bond 1: A rated; coupon rate of 6%; matures in 6 years and pays interest semiannually; currently selling for $850; duration is 5.16 years. Bond 2: A rated; coupon rate of 10%; matures in 8 years and pays interest semiannually; currently selling for $1,100; duration is 7.15 years. Which of these bonds would you recommend to George and why? I. Bond 1 because it has a higher yield to maturity than Bond 2 II,Bond 2 because its higher coupon rate gives it a superior total return to Bond 1 IIIBond 2 because it has a higher duration than Bond 1

I only

Which of the following are considered bond classifications for multisector bond funds? I.Foreign bonds II.High-dividend-paying common stocks III.Commodities

I only

Which of these is a correct justification for use of an investment in a client's portfolio? I.Blue chip common stocks because they provide a hedge against inflation II.FNMA (Federal National Mortgage Association) securities because they are backed by the full faith and credit of the U.S. government IIIAggressive growth stocks because they perform better during economic contractions

I only. Stocks generally are considered an inflation hedge; in periods of hyper-inflation, this may not be true, but the question does not ask about periods of hyper-inflation. FNMA securities are not backed by the full faith of the government (the government did step in as a result of the credit crisis of 2008, but there has not been a commitment to permanently back FNMAs in the same way that GNMAs (Government National Mortgage Association) have historically been backed).

Choose the CORRECT statements regarding option contracts. I.The buyer of a call option has the potential for unlimited gain. II.The seller of a put option is bullish. III.The writer of a naked call is exposed to an unlimited loss. IV.If the writer of a call option owns the underlying stock, the option is considered covered.

I, II, III, and IV

Identify those risks that pertain to hedge funds. I.Overuse of leverage II.Excessive short selling III.Lack of transparency IV. Lack of regulation

I, II, III, and IV

Which of these factors should an investor consider when investing in mutual funds? I.Fund performance II.Consistency of performance III.Management continuity IV. Fund age

I, II, III, and IV

Equity income funds may hold which of these types of securities? I.Income-producing common stocks II.Convertible bonds III. Convertible preferred stocks

I, II, and III

Which of the following methods can be used in determining the basis in a mutual fund when the shares were acquired at different times? I.Specific identification II.First in, first out (FIFO) III.Average cost method

I, II, and III

Which of the following statements concerning dollar cost averaging as a portfolio management technique are CORRECT? I. This technique involves investing a specific amount into an investment vehicle, regardless of whether the recent trend in the investment has been up or down. II. If prices decline, the fixed investment amount will purchase a greater quantity of the security. III. For the long-term investor, the presumption is that prices will eventually rise, so a lower average price translates into greater profits. IV. If prices rise, the fixed investment amount will purchase a greater amount of the security.

I, II, and III

Which of these statements concerning the principles of real estate investing is CORRECT? I.Investors in undeveloped land are seeking returns primarily from capital appreciation. II.Effective real estate investing almost always involves a careful but extensive use of leverage. III.Investors in real estate are usually seeking tax benefits for mortgage interest, property taxes, and depreciation. IV.Generally, investors in developed land experience significant taxable income in the early years of ownership

I, II, and III

ou are about to recommend international mutual funds to your clients. Which of the following are characteristics of investing internationally? I.International markets are less efficient than U.S. markets. II.International mutual funds have the exchange rate risks of individual foreign stocks. III.Due to lower correlations with U.S. stocks, foreign stocks can lower total portfolio risk. IV.Investors in foreign securities avoid U.S. tax on realized capital gains.

I, II, and III

Exchange-traded funds (ETFs) generally offer which of these? I.Tax efficiency II.Low expense ratios III.Active professional management IV. Marketability

I, II, and IV

Dave and Pam Larson, ages 65 and 63, respectively, recently retired. They successfully saved for their retirement throughout their careers using a low-risk approach. They would like to restructure their investments to have current income now to travel in their leisure time. Which of the following investment alternatives would be appropriate for the Larsons' goal? I.Equity income mutual fund shares II.Aggressive growth mutual fund shares III.Newly issued U.S. government bonds IV. GNMA fund shares

I, III, and IV

An investment portfolio has the following three stocks: StockInvestmentBetaStock 1$8,0000.6Stock 2$22,0001.3Stock 3$12,0000.9 Which of the following are CORRECT statements about this portfolio? IThe weighted beta for the portfolio is 0.93. IIThe weighted beta for the portfolio is 1.05. IIIThe portfolio is less risky than the market. IV.The portfolio is riskier than the market.

II and IV

Holly and Jeff are a married couple who have recently retired and are no longer earning an income. They would like to change their asset allocation to provide more income in their retirement years. Which of the following investments should be recommended to help the couple in achieving their financial objectives? I.Aggressive growth mutual fund II. AA rated corporate bonds III. Zero-coupon bonds IV. Equity income mutual fund

II and IV

Identify which of these methods may be used to trade exchange-traded funds (ETFs). I.Investors can buy or redeem shares from the fund family in lots of 1,000. II.Investors can trade ETFs in the secondary market by using a broker. III.ETFs can be purchased on margin. IV. ETFs may be sold short.

II, III, and IV

Which of the following statements assessing derivative securities is CORRECT? I.A call option is at the money when the exercise price is less than the market price of the stock. II.A put option is at the money when the exercise price is greater than the market price of the stock. III.Buyers of calls are betting that the price of the underlying common stock will rise, making the call option more valuable. IV.Put buyers are betting that the price of the underlying common stock will decline, making the put option more valuable.

III and IV

Which of these statements regarding unit investment trusts (UITs) are CORRECT? I.A bond UIT has a yield to maturity. II.UIT sponsors must make a secondary market in the UITs they create. III.UITs have management fees lower than mutual funds. IV. A bond UIT does not replace bonds that are called.

III and IV

A major source of systematic risk when investing in foreign securities is exchange rate risk. To minimize the effect of exchange rate risk, an investor can hedge with currency futures. Which of the following illustrate appropriate times for an investor who owns a foreign security to hedge? I.The foreign security is expected to rise in price and the dollar is expected to fall relative to the foreign currency. II.The foreign security is expected to remain relatively stable in price and the dollar is expected to fall relative to the foreign currency. III.The foreign security is expected to fall in price and the dollar is expected to rise relative to the foreign security. IV.The foreign security is expected to fall in price and the dollar is expected to remain relatively stable relative to the foreign security.

III only

Identify which of these statements regarding option pricing models is CORRECT. I.The binomial option pricing model assumes that the price of the option will change constantly because the market price of the underlying security also changes constantly. II.The Black-Scholes option valuation model is designed to determine the price of an American call option. III.The Black-Scholes option valuation model assumes that the price of the option will change in discrete increments on the basis of movements (up or down) in the price of the underlying stock. IV.The binomial model assumes the call option being valued has an exercise price of $100.

IV only

Wendy is concerned that her investment's actual return will not equal its expected return. Point out the type of risk that she is concerned about regarding her investment.

Investment risk

Options with expiration dates more than two years out are categorized as

LEAPS.

During his next meeting with his financial advisor, Zachary would like to compare the performance of his international investments against a benchmark. Select the appropriate benchmark to use for this comparison.

MSCI EAFE Index

The Galaxy Fund has a standard deviation of 15, and a mean return of 9%. The Universe Fund has a standard deviation of 22, and a mean return of 13%. The Milky Way Fund has a standard deviation of 18, and a mean return of 11%. Which fund should you choose in order to minimize the risk per unit of return?

Milky Way Fund

In a positively skewed distribution, what is the order (from lowest value to highest) for the distribution's mode, mean, and median values?

Mode, median, mean

LFM Corporation declared a record date of Wednesday, May 16, for its next quarterly cash dividend. Determine the last day an investor can purchase LFM stock and receive the current dividend.

Monday, May 14

Which of these investments is advisable during periods of rising interest rates?

Money market mutual funds. Short-term money market instruments are attractive during periods of rising interest rates. U.S. Treasury bills and negotiable CDs are also advisable. These investments do not experience significant erosion in value in response to increasing interest rates.

Due to an inheritance, Danielle now owns a large position in XYZ stock. She is concerned that the stock may decline in the upcoming months while she is deciding what to do with the investment. What type of investment strategy could her financial planner propose to protect the stock from substantial downside risk?

Purchase a put option

Michael owns a municipal bond, trading at par, with a 4.25% coupon rate and is in the 32% federal marginal income tax bracket. Calculate the taxable equivalent yield (TEY) for this bond.

The answer is 6.25%. TEY = tax-exempt yield ÷ (1 − marginal tax rate) = 0.0425 ÷ (1 − 0.32) = 0.0625, or 6.25%. Therefore, the TEY for Michael's bond is 6.25%. Michael would require this rate of return or higher for an equivalent taxable bond.

An investor who is interested in developing a portfolio of collectibles should probably do which one of these?

Specialize in a type of collectibles

Stock A has an expected mean return of 15% and a standard deviation of 22%; Stock B has an expected mean return of 11% and a standard deviation of 13%; and Stock C has an expected mean return of 18% and a standard deviation of 24%. You want to recommend one of these stocks to a client who is most interested in owning stocks that are more likely to deliver the expected mean return. Which stock should you recommend to meet this client's requirement?

Stock B. The coefficient of variation is a measure of the degree of variation of returns compared with the expected mean return. The security with the lowest coefficient of variation is the one most likely to deliver periodic returns closest to its expected return. The coefficients of variation of the three securities are 1.47 for Stock A, 1.18 for Stock B, and 1.33 for Stock C. Stock B should be recommended.

Consider this information regarding two possible investments: Stocks J and K. Stock J: Expected return: 11.5% Standard deviation: 8% Stock K: Expected return: 8.2% Standard deviation: 6% Identify which of these investments you would prefer and why.

Stock J because it has the lowest coefficient of variation

Assume an investor purchased $10,000 of Fund ABC at the beginning of Year 1. Subsequently, he made investments at the beginning of Years 2, 3, and 4 of $1,000, $5,000, and $8,000, respectively. At the beginning of Year 5, the fund was worth $33,000. What was the internal rate of return (IRR) on this fund?

The answer is 12.79%. This problem involves calculating the IRR/YR for uneven cash flows per the following inputs. Using the HP 10bII+: (10,000) CFj, (1,000) CFj (5,000) CFj (8,000) CFj 33,000 CFj SHIFT, IRR/YR = 12.79%

Frank purchased 100 shares of ABC common stock five years ago at a cost of $5,000. The stock paid the following dividends: YearAmount1-$200.00 2-$200.00 3-$250.00 4-$275.00 -5$300.00 At the time the fifth-year dividend was paid, Frank sold the stock for $8,500. What is the dollar-weighted return on ABC stock?

The answer is 15.11%. This problem involves calculating the IRR/YR for uneven cash flows per the following inputs. Using the HP 10bII+: 5000, +/-, CFj 200, CFj 200, CFj 250, CFj 275, CFj 8800 ($300 + $8,500), CFj SHIFT, IRR/YR = 15.1076

Anita bought ULA stock for $25,000 six years ago. Today, she sold the stock for $67,000. Calculate Anita's holding period return on ULA.

The answer is 168.00%. Anita's holding period rate of return was 168% {[(67,000 - 25,000) ÷ 25,000] × 100}.

Jonathan purchased 500 shares of CPM stock for $12 per share. At the end of the first year, he made another purchase of 500 shares at a stock price of $12 per share. At the end of the third year, he sold all of the stock for $17 per share. In addition, the stock paid a dividend of 0.35 per share at the end of each year. Calculate the dollar-weighted return to Jonathan over the three-year period.

The answer is 17.46%. Jonathan earned a dollar-weighted rate of return of 17.46% on CPM stock over the three-year period, calculated as follows: CF0 = −12 × 500 = −6,000 CF1 = (−12 × 500) + (0.35 × 500) = −5,825 CF2 = (0.35 × 1,000) = 350 CF3 = (0.35 × 1,000) + (17 × 1,000) = 17,350 Solve for the internal rate of return (IRR/YR) = 17.4626, or 17.46%

Carolyn owns a corporate bond with a coupon rate of 3.60%. Currently, the inflation rate is 1.50%. Calculate the real rate of return on this bond.

The answer is 2.07%. The real rate of return on this bond is calculated as follows: [((1 + 0.036) ÷ (1 + 0.015)) - 1] × 100 = 0.0207 × 100 = 2.07%

Johnny owns a municipal bond with a coupon rate of 4.25%. Assuming the annual inflation rate is 1.65%, calculate Johnny's real rate of return on his bond.

The answer is 2.56%. Johnny realized a real rate of return of 2.56%. Real rate of return = {[(1 + 0.0425) ÷ (1 + 0.0165)] - 1} × 100 = 2.5578, or 2.56%.

Myles purchased 1,000 shares of XYZ growth fund for $15 per share. At the end of the two years, he sold all of the shares for $22 per share. At the end of each year, the fund paid a dividend of $0.50 per share. Calculate the fund's time-weighted return over the two-year period.

The answer is 24.15%. The fund produced a 24.15% time-weighted rate of return over the two-year period, calculated as follows: CF0 = -15 × 1,000 = −15,000 CF1 = 0.50 × 1,000 = 500 CF2 = (0.50 × 1,000) + (22 × 1,000) = 22,500 Solve for the internal rate of return (IRR/YR) = 24.1525% (rounded to 24.15%)

Strahan Corporation's current annual common stock dividend is $3 and is expected to grow by 15% during the next year. The stock's current market price is $35 per share. If Strahan stock had a $30 market price one year ago, calculate the stock's total return.

The answer is 26.67%. A stock's total return (TR) = (the dividend received during a given period + the change in the stock's price during the same period) ÷ by the stock's current market value at the beginning of the period. Therefore, TR = ($3 + $5) ÷ $30 = 26.67%.

Leslie purchased a 10-year bond with a coupon rate of 4.75% paid semiannually. The bond has a current market price of $1,035. Calculate the yield to maturity (YTM) for Leslie's bond.

The answer is 4.3154%. The bond's YTM is calculated as follows: END Mode, 2 P/YR PV = −1035 FV = 1000 PMT = 4.75% × 1000 ÷ 2 = 23.75 10, DOWNSHIFT, N = 20 Solve for I/YR = 4.3154%

Eight years ago, ABC Company issued a 20-year bond with a 4% coupon rate. Due to a recent decline in market interest rates, the company decided to call the bonds for 103% of par value. Calculate the rate of return for an investor who purchased the bond at issue for par and surrendered it today for the call price.

The answer is 4.32%. The yield to call on this issue is calculated as follows: END Mode, 2 P/YR PV = -1000 FV = 1030 PMT = 4% x 1000 ÷ 2 = 20 8, DOWNSHIFT, N = 16 Solve for I/YR = 4.32%

Billie purchased a 10-year U.S. Treasury bond with a 6.5% coupon paid semiannually. Assuming the bond is currently trading at $1,075, calculate its yield to maturity (YTM).

The answer is 5.51%. END Mode, 2 P/YR PV = -1,075 FV = 1,000 PMT = 32.50 (6.5% × 1,000 ÷ 2) 10, DOWNSHIFT, N = 20 Solve for I/YR = 5.51%. The YTM on the bond is 5.51%, which is lower than the coupon rate of 6.5%, further validating that the bond is trading at a premium.

Andy purchased a four-year bond with a coupon rate of 7.5% paid semiannually. The bond is trading for $1,025 in the secondary market. Calculate the bond's yield to maturity (YTM).

The answer is 6.78%. The bond's YTM is calculated as follows: END Mode, 2 P/YR PV = -1,025 FV = 1,000 PMT = 37.50 (1,000 × 7.5% ÷ 2) 4, DOWNSHIFT, N = 8 Solve for I/YR = 6.78%.

You are choosing between two investments: Mutual Fund A with a return of 12% and a standard deviation of 18%, and Mutual Fund B with a return of 8% with a standard deviation of 12%. If the risk-free rate is 3%, which fund should you choose based upon one of the risk-adjusted return measurements?

The answer is Fund A. Enough information is given to calculate the Sharpe ratio for each investment. For Fund A (12 - 3) ÷ 18 = 0.50. For Fund B (8 - 3) ÷ 12 = 0.42. The higher the ratio, the better, so Fund A is the best choice.

Identify which of these statements regarding unit investment trusts (UITs) is CORRECT. I.Units are sold at net asset value plus a commission for the broker executing the transaction. II.Like stocks, UITs are traded on the major exchanges. III.During the term of the trust, unit holders are taxed in the same manner as owners of variable annuities. IV. Upon maturity, the securities are generally liquidated and the proceeds distributed to the investor or trust beneficiaries.

The answer is I and IV. Statements II and III are incorrect. UITs are sold in the secondary market, but not on the major exchanges. During the term of the trust, unit holders are taxed in the same manner as shareholders of mutual funds with capital gains earned by the trust passed through and taxed to the unit holders. Dividends are taxed as ordinary income in the year earned.

Adam is trying to evaluate the performance of his portfolio on a risk-adjusted basis. He has a nondiversified portfolio of large-cap stocks. He knows there are different measures of risk-adjusted performance and is not sure which one to use. Which of the following is the most appropriate measure to use?

The answer is Sharpe, because when a portfolio represents the entire investment fund, standard deviation is a better measure of risk. The Sharpe ratio must be computed for a benchmark, which is then compared to the performance of a portfolio. The Treynor ratio is used when the performance of a subportfolio is measured. Alpha is computed using the Jensen performance measure.

Dividend reinvestment plans offer which of these advantages?

The answer is a convenient means to accumulate shares. The advantage to an investor is the saving of commissions by using a dividend reinvestment plan (DRIP). The advantage is to an investor, not to the company. The use of a DRIP has no effect on the stock's par value. A dividend reinvestment program has no effect on the company's ability to retain more earnings.

When analyzing various investment alternatives, investors would generally choose which of these? A)An investment exhibiting a low positive skewness and a leptokurtic distribution B)An investment exhibiting a high positive skewness and a leptokurtic distribution C)An investment exhibiting a high positive skewness and a platykurtic distribution D)An investment exhibiting a low positive skewness and a platykurtic distribution

The answer is an investment exhibiting a high positive skewness and a leptokurtic distribution. Investments exhibiting high positive skewness have a larger than average number of positive price movements. Also, investments exhibiting a leptokurtic distribution have more observations clustered closely around the mean, resulting in a lower variance. Investors prefer a large number of positive returns with low risk.

Brandon owns ABC mutual fund that has produced the following returns over the past three years: Year 1: 4.7% Year 2: −10.0% Year 3: 6.5% Based on this information, calculate both the arithmetic mean (AM) and geometric mean (GM) returns for this series.

The answer is arithmetic mean: 0.40%; geometric mean: 0.1182%. The arithmetic mean is calculated as follows: (4.7% - 10.0% + 6.5%) ÷ 3. The geometric mean is calculated as follows: PV = -1, FV = (1.047)(0.90)(1.065), PMT = 0, N = 3, solve for I/YR = 0.1182.

A Japanese bank has decided to use some of its U.S. dollar reserves, resulting from the U.S. merchandise trade deficit with Japan, to invest in U.S. Treasury bonds. The U.S. Treasury securities pay approximately 6% interest, compared to 2% interest paid on Japanese bonds. For the Japanese bank to retain at least this differential in interest income, which of these situations in the foreign exchange market would have to occur?

The answer is the yen would have to depreciate or remain steady relative to the dollar during the holding period of the bond. When an investor in Country A invests in a security in Country B, the investor benefits if Country A's currency depreciates relative to Country B (or Country B's currency appreciates relative to Country A). Because rising interest rates cause bond prices to decline, the best possible scenario would be for the yen to depreciate and for U.S. interest rates to fall. This would give both a capital gain and a currency gain.

You are about to invest in foreign mutual funds and have decided to invest in country funds as opposed to a single diversified international mutual fund. Your single biggest concern is the fact that the U.S. dollar has been rising dramatically against the currency of this country. Which of the following points is most important?

The foreign currency should be fully hedged.

Mary owns a put option with an exercise price of $20 per share. The option is currently trading for $0.53 and the underlying stock is currently trading for $19.67 per share in the secondary market. Based on this information, select the INCORRECT statement.

The option is out-of-the-money..The answer is the option is out-of-the-money. The option is in-the-money because the option has a positive intrinsic value. The option has an intrinsic value of $0.33 ($20.00 - $19.67) and a time value of $0.20 ($0.53 - $0.33).

An investment, such as an option, whose value is based on that of another security is classified as

a derivative.

Select the repayment period that subjects the investor to least amount of interest rate risk.

Tranche A

Louis owns an investment that is an unmanaged portfolio in which the money manager initially selects the securities to be included in the portfolio and then holds those securities until they mature or the investment portfolio terminates. This statement best describes which type of investment?

Unit investment trust

Which of the following are characteristics of the Sharpe ratio? I. The ratio adjusts the return for variability by using standard deviation as the measure of risk. II.The ratio assumes that the portfolio being evaluated is well diversified. III.Both alpha and beta appear in the formula for the ratio. IV. The ratio indicates by how much the realized return differs from the return expected by the capital asset pricing model

Which of the following are characteristics of the Sharpe ratio? The ratio adjusts the return for variability by using standard deviation as the measure of risk. The ratio assumes that the portfolio being evaluated is well diversified. Both alpha and beta appear in the formula for the ratio. The ratio indicates by how much the realized return differs from the return expected by the capital asset pricing model

An investor should choose index funds if he or she believes in

a passive investment strategy.

To be on a corporation's books as holder-of-record (and thus have a right to the next dividend payment), the investor must purchase stock

before the ex-dividend date.

Which one of these factors has the greatest impact on the standard deviation of a two-asset portfolio? A)The standard deviation of each security in the portfolio B)Covariance C)The weight of each security in the portfolio D)The portfolio's beta

covariance. Covariance is the most important variable in minimizing the standard deviation of a portfolio. The weight and standard deviation are not as critical as the covariance of the two securities. Beta is not used in the formula to compute a portfolio's standard deviation.

Which of these types of risk is associated with the degree to which a company utilizes debt to finance its operations?Business risk; Financial risk;Default risk; Credit risk

financial risk. Financial risk is associated with the degree to which a company utilizes debt to finance its operations.

An investor who owns a sector fund that has substantial unsystematic risk and would like to know how a portfolio manager performed on a risk-adjusted basis would use which of the following indicators?

harpe uses standard deviation and assumes the portfolio is not well diversified and measures total risk.

Kumar purchased 100 shares of YTR stock from his broker. He earned returns of 7%, -3%, 8%, -10%, and 12% in years 1 through 5 respectively. Calculate Kumar's geometric mean return on his investment over the five-year period.

he answer is 2.47%. First, calculate the future value per $1 of Kumar's investment: (1.07)(0.97)(1.08)(0.90)(1.12) = 1.13. Next, calculate the geometric mean: PV = -1, FV = 1.13, N = 5, solve for I/YR = 2.4745, or 2.47%.

A distribution that is more peaked than normal is

leptokurtic.

Long-term bond funds have

more interest rate risk than short-term bonds.

An investor using a dollar cost averaging approach to buying a mutual fund will buy

more shares when the NAV of the fund falls since the last purchase.

capitalization-weighted indexes are

preferred type of index to use in modern portfolio theory applications. Capitalization weighted indexes are the most prevalent type of index and are best suited for modern portfolio theory applications. In a price-weighted index, higher-priced stocks within this index have more influence on the overall movement of this index than lower-priced stocks.

Unsystematic (unique) risk can be reduced by buying

stocks in numerous unrelated companies. Owning stock in unrelated companies results in holding stocks that have a low correlation coefficient between them. If a portfolio has numerous diversified issues of stocks, an investor can reduce and virtually eliminate the degree of unsystematic (unique) risk in the portfolio. Buying stocks in international companies can help to reduce systematic risk, because those stocks trade in different markets.

Tripp is an investor in the 32% marginal tax bracket. If he invests in a 4.75% municipal bond, his taxable equivalent yield (TEY) would be

taxable equivalent yield = tax-exempt yield ÷ (1 − marginal tax rate) = 4.75% ÷ (1 − 0.32) = 6.99%

Mark owns a corporate bond with a coupon rate of 6.78%. Assume the annual inflation rate is 2.5% and he is in the 35% federal marginal income tax bracket. Calculate his after-tax, inflation-adjusted rate of return on this bond.

the answer is 1.86%. First, calculate Mark's after-tax rate of return on the corporate bond [0.0678 × (1 - 0.35)] = 0.04407, or 4.41%. Next, calculate the after-tax, inflation-adjusted rate of return {[(1 + 0.0441) ÷ (1 + 0.025)] - 1} × 100 = 1.8634, or 1.86%.

Shannon is evaluating the absolute performance of the Shining Star mutual fund. The return of the fund for the past year was 13%, beta is 1.10, and standard deviation is 23. The market return is 9.5%, and the risk-free rate is 4.5%. Which of the following statements is true?

the fund's alpha is +3, meaning that the fund manager achieved a higher return than required for the amount of risk taken. Alpha is an absolute measure that is simply the difference between the return of the portfolio and the required return (CAPM). The formula is rp - [rf + (rm - rf)β. 13 - [4.5 + (9.5 - 4.5)1.1] = 3. Treynor and Sharpe are comparative or relative measures, and you would choose the investment with the highest number. Alpha is an absolute measure, giving you the actual return above the required return.

When using a security market index to represent a market's performance, the performance of that market over time is best represented by

the percent change in the index value.

Select the CORRECT statement regarding investors who only purchase high-beta stocks. A)They prefer stocks with high risk and high positive skewness. B)They prefer stocks with high risk and low positive skewness. C)They prefer stocks with low risk and high positive skewness. D)They prefer stocks with low risk and low positive skewness.

they prefer stocks with high risk and high positive skewness. Generally, these types of investors would prefer stocks with high risk (high beta) and high positive skewness that provide the opportunity for high rates of return. Stocks exhibiting high positive skewness have a larger than average number of positive price movements.

To be on a corporation's books as holder-of-record (and thus have a right to the next dividend payment), the investor must purchase stock

two business days before the record date.

When a company issues an option to buy its stock at a specified price within a specified time period, it is known as a

warrant.

The difference between a convertible bond and a bond with warrants is that

when warrants are exercised, the issuer receives equity capital from the warrants in addition to the original debt from the bond with warrants; when convertible bonds are converted, the bond is replaced with stock.


Conjuntos de estudio relacionados

HESI A2 MATH ON THE EXAM**, HESI A2 Math Questions, Hesi A2 V2 Math, HESI master sophia Ae

View Set

Chapter 1: Introduction to Organizational Management and Leadership

View Set

Chapter 2 - Nature of Insurance, Risk, Perils and Hazards

View Set

Anatomy: Bones of Lower Extremity

View Set

LIFESPAN DEVELOPMENT FINAL EXAM REVIEW

View Set

GLG 201 FINAL (all previous exams)

View Set

Chapter 7 - Cross-Cultural Communication and Negotiation

View Set